Last visit was: 25 Apr 2024, 12:01 It is currently 25 Apr 2024, 12:01

Close
GMAT Club Daily Prep
Thank you for using the timer - this advanced tool can estimate your performance and suggest more practice questions. We have subscribed you to Daily Prep Questions via email.

Customized
for You

we will pick new questions that match your level based on your Timer History

Track
Your Progress

every week, we’ll send you an estimated GMAT score based on your performance

Practice
Pays

we will pick new questions that match your level based on your Timer History
Not interested in getting valuable practice questions and articles delivered to your email? No problem, unsubscribe here.
Close
Request Expert Reply
Confirm Cancel
SORT BY:
Date
Tags:
Show Tags
Hide Tags
Senior Manager
Senior Manager
Joined: 07 Dec 2017
Posts: 325
Own Kudos [?]: 1664 [37]
Given Kudos: 348
GMAT 1: 650 Q50 V28
GMAT 2: 720 Q49 V40
Send PM
Most Helpful Reply
RC & DI Moderator
Joined: 02 Aug 2009
Status:Math and DI Expert
Posts: 11178
Own Kudos [?]: 31928 [9]
Given Kudos: 290
Send PM
GMAT Club Verbal Expert
Joined: 13 Aug 2009
Status: GMAT/GRE/LSAT tutors
Posts: 6921
Own Kudos [?]: 63668 [5]
Given Kudos: 1774
Location: United States (CO)
GMAT 1: 780 Q51 V46
GMAT 2: 800 Q51 V51
GRE 1: Q170 V170

GRE 2: Q170 V170
Send PM
General Discussion
Senior Manager
Senior Manager
Joined: 07 Dec 2017
Posts: 325
Own Kudos [?]: 1664 [1]
Given Kudos: 348
GMAT 1: 650 Q50 V28
GMAT 2: 720 Q49 V40
Send PM
Re: If Skiffs book is published this year, Professor Nguyen [#permalink]
1
Kudos
chetan2u wrote:
AshutoshB wrote:
If Skiffs book is published this year, Professor Nguyen vows she will urge the dean to promote Skiff. Thus, if Skiff's book is as important and as well written as Skiff claims, he will be promoted, for Nguyen will certainly keep her promise, and the dean will surely promote Skiff if Nguyen recommends it."

The argument's conclusion can be properly inferred if which one of the following is assumed?"

(A) Skiffs book will be published this year if it is as important as he claims it is."

(B) Skiff needs to publish a book before he can be promoted."

(C) Professor Nguyen believes that Skiff's book is well written."

(D) Skiffs book will not be published unless it is as important and as well written as he claims it is."

(E) Skiff will not be promoted unless Professor Nguyen urges the dean to do so.

LSAT



9 people out of 10 going wrong means there is a bit of confusion when it comes to IF construction...

Para..
If S's book published, D will urge for S's promotion...
If S's book is as important as he claims he will get promoted as D will urge and Dean will surely act on D's recommendations.

As soon as you read it, you should find the gap..
The importance of the book is directly linked to recommendations but recommendations were base on the fact that book gets published..
So there is a gap between the importance and the publishing..

Choice A touches upon this very issue..
If the book is as important as S claims, it will get published

A



Great explanation as always +1
RC & DI Moderator
Joined: 02 Aug 2009
Status:Math and DI Expert
Posts: 11178
Own Kudos [?]: 31928 [2]
Given Kudos: 290
Send PM
Re: If Skiffs book is published this year, Professor Nguyen [#permalink]
2
Kudos
Expert Reply
AshutoshB wrote:
chetan2u wrote:
AshutoshB wrote:
If Skiffs book is published this year, Professor Nguyen vows she will urge the dean to promote Skiff. Thus, if Skiff's book is as important and as well written as Skiff claims, he will be promoted, for Nguyen will certainly keep her promise, and the dean will surely promote Skiff if Nguyen recommends it."

The argument's conclusion can be properly inferred if which one of the following is assumed?"

(A) Skiffs book will be published this year if it is as important as he claims it is."

(B) Skiff needs to publish a book before he can be promoted."

(C) Professor Nguyen believes that Skiff's book is well written."

(D) Skiffs book will not be published unless it is as important and as well written as he claims it is."

(E) Skiff will not be promoted unless Professor Nguyen urges the dean to do so.

LSAT



9 people out of 10 going wrong means there is a bit of confusion when it comes to IF construction...

Para..
If S's book published, D will urge for S's promotion...
If S's book is as important as he claims he will get promoted as D will urge and Dean will surely act on D's recommendations.

As soon as you read it, you should find the gap..
The importance of the book is directly linked to recommendations but recommendations were base on the fact that book gets published..
So there is a gap between the importance and the publishing..

Choice A touches upon this very issue..
If the book is as important as S claims, it will get published

A



Great explanation as always +1



Great question too +1 for the question
Manager
Manager
Joined: 24 Dec 2018
Posts: 102
Own Kudos [?]: 84 [2]
Given Kudos: 35
Concentration: Entrepreneurship, Finance
Send PM
Re: If Skiffs book is published this year, Professor Nguyen [#permalink]
2
Kudos
chetan2u,

Why not D instead of A?
Intern
Intern
Joined: 25 Mar 2017
Posts: 9
Own Kudos [?]: 9 [0]
Given Kudos: 65
Send PM
Re: If Skiffs book is published this year, Professor Nguyen [#permalink]
Can someone help me understand why D is not the right answer?

Option A says the book will be published if it's important whereas the premise has two conditions: important and well written.
avatar
Intern
Intern
Joined: 16 Mar 2019
Posts: 1
Own Kudos [?]: 0 [0]
Given Kudos: 1
Send PM
Re: If Skiffs book is published this year, Professor Nguyen [#permalink]
Hello, could you please explain the OA?
Manager
Manager
Joined: 17 Aug 2018
Posts: 119
Own Kudos [?]: 156 [0]
Given Kudos: 150
Location: India
Schools: IIMA WBS '22
GMAT 1: 640 Q46 V32
GMAT 2: 710 Q49 V38
Send PM
Re: If Skiffs book is published this year, Professor Nguyen [#permalink]
NiruSinghal wrote:
Can someone help me understand why D is not the right answer?

Option A says the book will be published if it's important whereas the premise has two conditions: important and well written.


I also have exactly the same doubt.

GMATNinja. Please share your two cents.
Manager
Manager
Joined: 17 Aug 2018
Posts: 119
Own Kudos [?]: 156 [0]
Given Kudos: 150
Location: India
Schools: IIMA WBS '22
GMAT 1: 640 Q46 V32
GMAT 2: 710 Q49 V38
Send PM
If Skiffs book is published this year, Professor Nguyen [#permalink]
GMATNinja wrote:
KaranB1 wrote:
NiruSinghal wrote:
Can someone help me understand why D is not the right answer?

Option A says the book will be published if it's important whereas the premise has two conditions: important and well written.


I also have exactly the same doubt.

GMATNinja. Please share your two cents.

Take a look at the exact wording of the answer choice:
Quote:
The argument's conclusion can be properly inferred if which one of the following is assumed?

So, we need an assumption that, combined with the evidence from the passage, allows us to properly infer the conclusion. Note: you will not see this form of question on the GMAT (where assumption questions will be some form of "the argument depends on which assumption?") Proceed only if this question has piqued your curiosity, otherwise your time is better spent on questions that align more closely to the GMAT.

Let's break down the passage to see where that assumption needs to fit in:

  • Fact 1: If Skiff's book is published this year, Prof. Nguyen will urge the dean to promote Skiff.
  • Fact 2: Prof. Nguyen will keep her promise, and the dean will promote Skiff if Nguyen recommends it
  • INSERT ASSUMPTION HERE
  • Condition on the conclusion: If Skiff's book is as important and well written as he claims,
  • Conclusion: Skiff will be promoted.

We are looking for an answer choice that, when inserted in the passage as described above, allows us to infer that the conclusion follows.

Let's first look at (D):
Quote:
(D) Skiffs book will not be published unless it is as important and as well written as he claims it is.

This tells us ONE reason why Skiff's book will not be published, but it does not tell us that it is the ONLY reason that his book may not be published. An infinite amount of other things could happen to prevent Skiff from publishing his book -- maybe his manuscript will be burnt to a crisp in a housefire, or maybe he'll decide to leave academia and take up subsistence farming on some remote island.

If one of these other scenarios occurs, then Skiff STILL will not be promoted, even if we assume the truth of (D). So, (D) does not GUARANTEE that the conclusion follows logically from the evidence in the passage. For this reason, (D) is out.

Compare that to (A):
Quote:
(A) Skiffs book will be published this year if it is as important as he claims it is.

Remember, Professor Nguyen doesn't care whether Skiff's book is important and well written -- If it is published this year, she will urge the dean to promote Skiff, and the dean will promote him. To get this chain of events to occur, we just need to prove that Skiff's book will be published.

(A) tells us that the book will be published if it is "as important as he claims it is." So, if it reaches that bar, the following will occur: the book will be published, Nguyen will pester the dean, and Skiff will be promoted.

The passage adds another criterion on to this list: Skiff will get promoted if the book is not only important, but also well written.

But remember, Nguyen doesn't care about these factors! If we assume (A), then we KNOW that if the book is important it will be published, and the rest of the chain of events will occur. This is a LOWER bar than assuming that the book is both important and well written. So, if Skiff will be promoted NO MATTER WHAT if he writes an important book, then he will certainly be promoted if the book is both important AND well written.

In other words, (A) fills in the gap between the evidence and conclusion in the passage -- if (A) is assumed, then the conclusion follows. The same can not be said for (D) -- if (D) is assumed, the conclusion might follow, but it also might not.

(A) is the correct answer.

I hope that helps!


GMATNinja Sir, I'm completely clear now as to why answer choice D is wrong.

My understanding is "Important and well written" is higher bar. Therefore, If this condition is met, event will surely occur. At times event may occur even when this condition is not met.

It is like if someone scores in "1st class overall (Condition 1) and 100% in maths(Condition 2) " in an examination, then that person is definitely pass. Someone may also pass without complying to the above conditions. So, Answer choice D is clearly wrong.

However, I'm not still not clear with the logic of answer choice A. This choice says one of the conditions is met and event will occur.

From the above example, I cannot say that if someone has scored 100% in maths, then that person has passed the exam. What if he/she failed in all other subjects.
Intern
Intern
Joined: 27 Feb 2019
Status:Active
Affiliations: NA
Posts: 23
Own Kudos [?]: -3 [1]
Given Kudos: 229
Location: India
GPA: 3.8
WE:Business Development (Manufacturing)
Send PM
Re: If Skiffs book is published this year, Professor Nguyen [#permalink]
1
Kudos
If Skiffs book is published this year, Professor Nguyen vows she will urge the dean to promote Skiff. Thus, if Skiff's book is as important and as well written as Skiff claims, he will be promoted, for Nguyen will certainly keep her promise, and the dean will surely promote Skiff if Nguyen recommends it."

The argument's conclusion can be properly inferred if which one of the following is assumed?"

(A) Skiffs book will be published this year if it is as important as he claims it is."

(B) Skiff needs to publish a book before he can be promoted."

(C) Professor Nguyen believes that Skiff's book is well written."

(D) Skiffs book will not be published unless it is as important and as well written as he claims it is."

(E) Skiff will not be promoted unless Professor Nguyen urges the dean to do so.

Ans- I will try to give me view, hope it helps anyone looking for simple approach.

Conclusion goes like this ---> If skiffs book is to be published then --> Professor Nguyen will definitely urge to promote him (he promised shows this) --> which leads to dean to promote him (he will only promote him if prof. gives his support). So in order to get promotion Skiffs need to publish that book and that too in this year (note: we are not sure about next year, we are given information on this year only).

Now read sentence A it will show this is necessary assumption to have the conclusion. Also,you can try to negate it and get the answer.

Negated statement - "Skiffs book will NOT be published this year if it is as important as he claims it is." destroy the conclusion. Hence this is the answer.
GMAT Club Verbal Expert
Joined: 13 Aug 2009
Status: GMAT/GRE/LSAT tutors
Posts: 6921
Own Kudos [?]: 63668 [1]
Given Kudos: 1774
Location: United States (CO)
GMAT 1: 780 Q51 V46
GMAT 2: 800 Q51 V51
GRE 1: Q170 V170

GRE 2: Q170 V170
Send PM
Re: If Skiffs book is published this year, Professor Nguyen [#permalink]
1
Kudos
Expert Reply
KaranB1 wrote:
GMATNinja Sir, I'm completely clear now as to why answer choice D is wrong.

My understanding is "Important and well written" is higher bar. Therefore, If this condition is met, event will surely occur. At times event may occur even when this condition is not met.

It is like if someone scores in "1st class overall (Condition 1) and 100% in maths(Condition 2) " in an examination, then that person is definitely pass. Someone may also pass without complying to the conditions. So, Answer choice D is clearly wrong.

I'm not still not clear with the logic of answer choice A. This choice says one of the conditions is met and event will occur.

From the above example, I cannot say that if someone has scored 100% in maths, then that person has passed the exam. What if he/she failed in all other subjects.

Referring to your example, let's say that you WILL pass the class if you scored 100% in maths. In that case, the following is a true statement: "If you score 100% in maths, are first in your class, and run a mile in under 6 minutes, then you will pass the class." Sure, the last two things weren't necessary at all, but if you've done all three, then you will definitely pass.

Similarly, (A) tells us that IF the book is as important as Skiff claims it is, then Skiff's book WILL be published this year. It follows that if the book is both 1) as important as Skiff claims it is and 2) as well-written as Skiff claims it is, then Skiff's book will be published this year. The part about being well-written wasn't necessary, but if the well-written condition AND the (required) importance condition are met, then the book will be published this year.

As long as Skiff's book is as important as he claims it is, the book will be published and thus Skiff will be promoted. So it would be correct to conclude, for example, that, "if Skiff's book is as important, as well written, as funny, as entertaining, {etc.}, as Skiff claims, he will be promoted." We only need the part about being important. Everything else is extra, but that doesn't make the conclusion inaccurate.

Shubhangana wrote:
What is wrong with B?

Quote:
(B) Skiff needs to publish a book before he can be promoted.

Let's review the logic here:

  • If Skiff's book is published, Professor Nguyen will urge the dean to promote Skiff.
  • If Nguyen urges the dean to promote Skiff, "the dean will surely promote Skiff."

So, if Skiff's book is published, he will be promoted. But is that necessarily the ONLY way Skiff will be promoted? Maybe Skiff will also be promoted if he makes a major research breakthrough, or if he wins the Faculty Member of the Year award, if he bribes the dean with cookies, etc. (Mmm... cookies.)

Publishing his book is one way to get promoted (it's a SUFFICIENT condition), but it's not necessarily the only way to get promoted (it's not a NECESSARY condition).
Intern
Intern
Joined: 07 Aug 2017
Posts: 12
Own Kudos [?]: 2 [0]
Given Kudos: 62
Send PM
Re: If Skiffs book is published this year, Professor Nguyen [#permalink]
Isnt the "recommendation" part more important, because irrespective of whether or not it is as important book , final decision will be triggered from recommendation. please advise how to counter such confusion
Manager
Manager
Joined: 17 Aug 2018
Posts: 119
Own Kudos [?]: 156 [0]
Given Kudos: 150
Location: India
Schools: IIMA WBS '22
GMAT 1: 640 Q46 V32
GMAT 2: 710 Q49 V38
Send PM
If Skiffs book is published this year, Professor Nguyen [#permalink]
P@PU wrote:
Isnt the "recommendation" part more important, because irrespective of whether or not it is as important book , final decision will be triggered from recommendation. please advise how to counter such confusion


P@PU

Lets first understand the flow.

(1)Skiff's book is important and as well written as Skiff claims > (2) Professor Nguyen recommends the book> (3) Skiff is promoted

From the Prompt we know if event 1 happens event 2 and 3 are sure to happen.

In if clauses you can always go from left to right, but you may or may not move right to left. Lets take an example to simplify this.

E.g.
If someone has scored 100% in the entrance exam, then he/she will get admission.

From the above example we need to know 100% score guarantees admission.
And admission does not always require 100% score.

Therefore, in the CR question at hand recommendation is not the highest in the bar order.
Intern
Intern
Joined: 24 Oct 2019
Posts: 5
Own Kudos [?]: 1 [0]
Given Kudos: 18
Send PM
Re: If Skiffs book is published this year, Professor Nguyen [#permalink]
A 100% certain promotion would be achieved if each of the following events occurs (a sequence):

1) Skiff's book is as important and as well written as Skiff claims

2) Skiffs book will be published this year (parts of answer A)

3) Professor Nguyen vows she will urge the dean to promote Skiff. (As his book has been published)

4) Nguyen will certainly keep her promise

5) The dean will surely promote Skiff if Nguyen recommends it.

Now, this is simply an ideal outcome: it has a 100% chance of suceeding.

BUT, we have other outcomes, which have a less-than-100 chance of suceeding. These outcomes are our answer choices. Let's look at them:

A) The correct answer

B) Skiff needs to publish a book before he can be promoted. --- not necessarily, he can still be promoted either way. The passage simply gives us a sure-fire way to promotion.

(C) Professor Nguyen believes that Skiff's book is well written. --- not necessary, either. She might or might not think so. All she cares about is to see the book be promoted, which (presumably) means other people like it. Doesn't mean Professor Nguyen has to like it.

(D) Skiffs book will not be published unless it is as important and as well written as he claims it is." --- It may still be published, even though it is less important than HE thinks it is. Other people might really like the book and want to see it published.

(E) Skiff will not be promoted unless Professor Nguyen urges the dean to do so. --- He may still be promoted. Again, Professor Nguyen's recommendation simply seals the deal. It does not mean that without the recommendation publication is impossible.

Posted from my mobile device
GMAT Club Bot
Re: If Skiffs book is published this year, Professor Nguyen [#permalink]
Moderators:
GMAT Club Verbal Expert
6921 posts
GMAT Club Verbal Expert
238 posts
CR Forum Moderator
832 posts

Powered by phpBB © phpBB Group | Emoji artwork provided by EmojiOne